Number Theory: The Euclidean Algorithm Proof

Поделиться
HTML-код
  • Опубликовано: 21 окт 2024
  • We present a proof of the Euclidean algorithm.
    www.michael-pen...

Комментарии • 67

  • @XxXMrGuiTarMasTerXxX
    @XxXMrGuiTarMasTerXxX 4 года назад +53

    I used to watch you for fun, now you're also helping me with my degree in maths :D Thank you!

  • @georgesadler7830
    @georgesadler7830 2 года назад +10

    Professor Penn, thank you for a classic proof on the Euclidean Algorithm. These are historic topics in mathematics.

  • @paigecarlson1742
    @paigecarlson1742 11 месяцев назад

    The way the last pos remainder percolates up. I appreciate finally explaining why it works, not just that it works.

  • @sporadicdrive5884
    @sporadicdrive5884 4 года назад +14

    Just found this channel and I'm loving the content so far! Thanks a lot

    • @subid.majumdar
      @subid.majumdar 3 года назад

      Do you know other channel like this, I want to learn more math

  • @Avighna
    @Avighna Год назад +3

    Wow! Amazing! I actually understood this (after a lot of effort). Always love the eureka moments I get when learning about a new proof. Thank you!

  • @mharouny1
    @mharouny1 6 месяцев назад +2

    The last part is because d

    • @kylebryancagasan4447
      @kylebryancagasan4447 2 месяца назад

      If I am not mistaken, that's because since the sequence r1, r2, ... , r_n-1 is decreasing and that d | r_n-1 implies d

  • @khizaramjad9620
    @khizaramjad9620 5 лет назад +16

    Well organized.
    Thanks alot!

  • @TolgaYilmaz1
    @TolgaYilmaz1 4 года назад +20

    Was it a typo at 5:25?
    You wrote:
    d | b - r1 * q1 implies d | r2
    I think it should have been:
    d | b - r1*q2 implies d | r2
    Since equation 2 is:
    b = r1*q2 + r2

  • @adinaadina4549
    @adinaadina4549 3 года назад +8

    Hi, I'm afraid you have so many views no one would answer my question. But still.
    I don't understand how you can deduce that d | a -bq from d | a, d | b. Can someone explain me the intermediate steps ?

    • @bingqingwang850
      @bingqingwang850 3 года назад +6

      In the previous "Divisibility Basic" part, there is a propositon that " If a|b and a|c, then for any x and y as integers, there is a|bx+cy". So according to this, we have d|a and d|b, then d|ax+by where x = 1, y = -q1.

    • @adinaadina4549
      @adinaadina4549 3 года назад +1

      @@bingqingwang850 okay thank you very much !!!

    • @mominahmed7-044
      @mominahmed7-044 3 года назад +4

      if d|a and d|b then we can write this as a = dx and b = dy (all integers). Any linear combination like pa + qb (p and q integers, again) can be written as dxa + dyb which you can factorize to d(xa + yb) which of course would be a multiple of d, the thing we initially sought to prove.

    • @KRITHEESH-it7un
      @KRITHEESH-it7un Год назад

      ​@@mominahmed7-044 thank you very much

  • @shuttlecrab
    @shuttlecrab Год назад

    Thank you for opening with what q and r are and how you get them.
    A good few articles I've read don't actually say what they are, which is annoying, because for a number-phobe like myself, a more granular introduction is needed quite often.
    For me, just looking at the equation does not intuitively tell me that q and r are generated by dividing a and b

  • @PunmasterSTP
    @PunmasterSTP 3 года назад +3

    Euclidean? More like Eu-clear-ian, because now I understand; thank you so much for explaining!

    • @MonkeyAssassin-qi8gw
      @MonkeyAssassin-qi8gw 3 месяца назад +1

      Great job punmaster, you’ve successfully ruined my day😂

    • @PunmasterSTP
      @PunmasterSTP 3 месяца назад

      @@MonkeyAssassin-qi8gw Then my work here is done 👍
      On the flip side, if you keep listening to painful puns, eventually you’ll become numb(er)…

    • @MonkeyAssassin-qi8gw
      @MonkeyAssassin-qi8gw 3 месяца назад +1

      @@PunmasterSTP you’re ruthless

    • @PunmasterSTP
      @PunmasterSTP 3 месяца назад

      @@MonkeyAssassin-qi8gw I once told someone something ridiculous, and they said "Euclid-en' me!"

  • @savi4420
    @savi4420 2 года назад

    How cutely u just proved it and i didn't even realised that I understood. Great teacher. Thankyou so much.

  • @krishshah3974
    @krishshah3974 4 года назад +1

    I don't know if this was intentional but the outfit and the background really match each other (ninja-like) :)

  • @Arjunsiva
    @Arjunsiva 3 года назад +4

    Awww, i miss the sound of writing on chalk board

  • @PMe-my1td
    @PMe-my1td 2 года назад +14

    Thanks for confirming how dumb I am 🙂

  • @khetnhio
    @khetnhio 4 года назад +2

    great proof, very clearly shown 10/10

  • @bettyswunghole3310
    @bettyswunghole3310 3 года назад +2

    Is this a proof by induction?

  • @souverain1er
    @souverain1er 3 года назад +2

    @Michael Penn How do you prove that the last sequence in the division has a remainder of zero?

    • @chinmayelastname3544
      @chinmayelastname3544 3 года назад +2

      that's because if the remained is not zero, then the sequence will keep going on

    • @yoman6367
      @yoman6367 3 месяца назад

      had the same question

  • @MrAmangandhi
    @MrAmangandhi 3 года назад +3

    at 5:41 shouldn't it be d divides b-(r1q2)

    • @c._olm3o945
      @c._olm3o945 6 месяцев назад

      Yes I think so but either way the point is clear I guess

  • @chitranshuvarshney4863
    @chitranshuvarshney4863 2 года назад

    well organized video

  • @rainmakesvideos
    @rainmakesvideos 4 года назад +2

    Does d | r_(n-1) imply (r_(n-1)) / d = 1 and that d = r_(n-1) ?
    Read: Does the last statement that d is a divisor of the last remainder imply that the remainder divided by d is equal to 1, and does that mean that d is equal to the last remainder?

    • @MichaelPennMath
      @MichaelPennMath  4 года назад +1

      Here we are using the definition of the gcd. We start by showing that r_{n-1} is a common division of a and b then suppose we have another common divisor of a and b and show that must divide r_{n-1}. This is precisely the definition of gcd(a,b). That d in the proof is only a divisor of the gcd, it may not be the gcd itself.

    • @xRabidz
      @xRabidz 4 года назад +4

      d is assumed to be ANY common divisor of a and b, so if d | r_(n-1) then that means d

    • @mateuszpachulski6211
      @mateuszpachulski6211 Год назад +1

      @@xRabidz I hope you see this comment :P As it drives me nuts :S we know that r__(n-1) is some common divisor, we do not know by the proof that it is the greatest one it may not be. So we take some divisor d, it may be the greatest but it also may not be the greatest, if we would take a d that is not the greatest one and r_(n-1) may not be the greatest one we are left with a d equal to r_(n-1), how does this proof that r_(n-1) is the greatest one ...

    • @theblinkingbrownie4654
      @theblinkingbrownie4654 Год назад +1

      ​@@mateuszpachulski6211r_(n-1) MUST be the greatest, we are not trying to prove with contradiction here. Basically gcd(a,b) is defined that ANY divisor d of a and b must divide the gcd. Think about it, any common factors of 30 and 40 must divide 10.

    • @arkashvijayakumar4181
      @arkashvijayakumar4181 11 месяцев назад +1

      thanks helped alot@@xRabidz

  • @G0j0_Satur0HI
    @G0j0_Satur0HI Месяц назад

    Hi can someone pls explain that if dIrn-1 how is it the gcd(a,b)?

  • @KFP4488
    @KFP4488 4 года назад +2

    How can we tell the sequence of r1, r2, r3 is decreasing?

    • @aaronzoll5289
      @aaronzoll5289 4 года назад +5

      By the division algorithm, a = bq + r where r < b because if it is great than or equal, it gets absorbed into q. So each r(n) < r(n-1)

  • @chickensalad1369
    @chickensalad1369 3 года назад

    yea I get this and its great and all but WHERE IS THE "And thats a good place to stop"

  • @VictorHugo-xn9jz
    @VictorHugo-xn9jz 9 месяцев назад

    Nice one.

  • @ashishkumarsharma5038
    @ashishkumarsharma5038 5 месяцев назад +1

    what if d=1?

    • @MonkeyAssassin-qi8gw
      @MonkeyAssassin-qi8gw 3 месяца назад

      Then there would be no purpose in proving😅,we assume that d does not equal one as there would be no point.

  • @footballistaedit25
    @footballistaedit25 4 года назад

    Thanks for a great video♥️♥️♥️

  • @nicholasalkhawli6057
    @nicholasalkhawli6057 4 года назад

    Shouldnt we prove that r1 and all other r don't divide a and b

  • @ignassiaulys9160
    @ignassiaulys9160 4 года назад +1

    wait so how is rn-1 a divisor of rn-3 if rn-2 isnt a divisor of rn-3?

    • @marceorigoni6614
      @marceorigoni6614 4 года назад

      rn-2 is not a divisor of rn-3, because rn-3/ rn-2 have a rest ≠ 0. I mean an example is 12 = rn-3 ,8 = rn-2 and 4 = rn-1. There you see 12/8 = 1*8 + 4 = 1,5. Not a divisor.

    • @gajrajsingh51
      @gajrajsingh51 3 года назад

      We know, r_{n-1} | r_{n-2}, hence q_{n-2} * r_{n-2} + r_{n-1} | r_{n-3}.

  • @chritophergaafele8922
    @chritophergaafele8922 4 года назад +3

    How do you prove that the sequence r1,r2,r3...rn decreases

    • @jackmaibach8316
      @jackmaibach8316 4 года назад +8

      r1 < b otherwise the choice for q1 was wrong.
      r2 < r1 otherwise the choice for q2 was wrong.
      and so on and so forth.
      this is a consequence of how remainder upon division is defined.

  • @siddhantkhare2775
    @siddhantkhare2775 2 года назад +3

    When did Mark Zuckerberg start teaching Mathematics.....Lmao🤣🤣🤣

  • @famoustrends_mmr
    @famoustrends_mmr 6 месяцев назад

    3:08 how to rn-1 divide rn-2 explain

    • @rayzewis3361
      @rayzewis3361 6 месяцев назад

      Because we can write r_{n-2} = r_{n-1}*q_{n}, which is equivalent to saying r_{n-1}|r_{n-2} (by divisibility definition: a=bn then a|n)

    • @famoustrends_mmr
      @famoustrends_mmr 6 месяцев назад

      @@rayzewis3361 but r_{n-2} = r_{n-1}*q_{n} + r_{n} sir you can use it not this r_{n-2} = r_{n-1}*q_{n}

  • @saikiranbanti5415
    @saikiranbanti5415 4 года назад

    Really a very good one easy to understand

  • @sakethram538
    @sakethram538 4 года назад

    if d/rn-1 then d should be the gcd of a & b

    • @keinernichts3531
      @keinernichts3531 4 года назад +2

      It's greatest common divisor, emphasis on greatest.

  • @zhuolovesmath7483
    @zhuolovesmath7483 2 года назад

    Take a rest buddy, you seem pretty tired

  • @brunon.8962
    @brunon.8962 4 года назад

    Why can't you prove this shit works by using real world things like a field or something useful and meaningful? Why does it have to be bulk lines of gibberish simbols?

  • @ibrahimElKhalil55
    @ibrahimElKhalil55 Год назад

    at 5:41 shouldn't it be d divides b-(r1q2)